Yo help me real quick?

Yo Help Me Real Quick?
Yo Help Me Real Quick?

Answers

Answer 1

Answer:

1,2 and 6

Step-by-step explanation:

pie symbol

2/3

0.333333....


Related Questions

Given the following hypotheses: H0: μ = 490 H1: μ ≠ 490 A random sample of 15 observations is selected from a normal population. The sample mean was 495 and the sample standard deviation 9. Using the 0.01 significance level:
a.) State the decision rule.
b.) Compute the value of the test statistic.
c.) What is your decision regarding the null hypothesis?

Answers

Answer:

We conclude that the population mean is equal to 490.

Step-by-step explanation:

We are given that a random sample of 15 observations is selected from a normal population. The sample mean was 495 and the sample standard deviation 9.

Let [tex]\mu[/tex] = population mean.

So, Null Hypothesis, [tex]H_0[/tex] : [tex]\mu[/tex] = 490      {means that the population mean is equal to 490}

Alternate Hypothesis, [tex]H_A[/tex] : [tex]\mu\neq[/tex] 490     {means that the population mean is different from 490}

The test statistics that will be used here is One-sample t-test statistics because we don't know about population standard deviation;

                               T.S.  =  [tex]\frac{\bar X-\mu}{\frac{s}{\sqrt{n} } }[/tex]  ~ [tex]t_1_4[/tex]

where, [tex]\bar X[/tex] = sample mean = 495

            s = sample standard deviation = 9

             n = sample of observations = 15

So, the test statistics =   [tex]\frac{495-490}{\frac{9}{\sqrt{15} } }[/tex]  ~ [tex]t_1_4[/tex]

                                     =  2.152

The value of t-test statistics is 2.152.

Now, at a 0.01 level of significance, the t table gives a critical value of -2.977 and 2.977 at 14 degrees of freedom for the two-tailed test.

Since the value of our test statistics lies within the range of critical values of t, so we have insufficient evidence to reject our null hypothesis as the test statistics will not fall in the rejection region.

Therefore, we conclude that the population mean is equal to 490.

normal population has a mean of 63 and a standard deviation of 13. You select a random sample of 25. Compute the probability that the sample mean is: (Round your z values to 2 decimal places and final answers to 4 decimal places): Greater than 65.

Answers

Answer:

0.2207

Step-by-step explanation:

Here, we want to find the probability that the sample mean is greater than 25.

What we use here is the z-scores statistic

Mathematically;

z-score = (x-mean)/SD/√n

From the question;

x = 65, mean = 63, SD = 13 and n = 25

Plugging these values in the z-score equation, we have

Z-score = (65-63)/13/√25 = 2/13/5 = 0.77

So the probability we want to calculate is ;

P(z > 0.77)

This can be obtained from the standard normal distribution table

Thus;

P(z > 0.77) = 0.22065 which is 0.2207 to 4 d.p

the coefficient of 6x

Answers

Answer:

The coefficient is 6

Step-by-step explanation:

The coefficient is the number in front of the variable

The variable is x

The coefficient is 6

Answer:

6

Step-by-step explanation: The coefficient of this would be the real number that is in front of a variable that is not a variable like x, and that number is 6. So, the coefficient of 6x is 6.

Please answer quick!!! Find the range of the data set represented by this box plot.

80
76
40
56

Answers

Answer:

highest value (H)= 80

lowest value (L)= 40

range (R)=?

now using formula,

Range (R)=H-L

=80-40

=40

therefore range (R)=40

Which of the following correlation values represents a perfect linear relationship between two quantitative
variables? Select all that apply.
A. 0
B. 9
c. -1
D. 1
E. .5

Answers

Answer:

  C.  -1

  D.  1

Step-by-step explanation:

A perfect linear relationship is indicated by a correlation with a magnitude of 1. The sign of the correlation coefficient is the sign of the slope of the line describing the relationship. It may be positive or negative.

The appropriate choices are ...

  C.  -1

  D.  1

Answer:

c=-1

d=1

Step-by-step explanation:

13,226 divided by 29

Answers

13226/29= 456.068965517

A parent increases a child’s monthly allowance by 20% each year. If the allowance is $8 per month now, in about how many years will it take to reach $20 per month? Use the equation 20 = 8(1.2)x to solve the problem. Round to the nearest year. 1 year 5 years 2 years 16 years

Answers

Answer:

6 years

Step-by-step explanation:

A parent increases a child’s monthly allowance by 20% each year. If the allowance is $8 per month now. This is an exponential function, An exponential function is given by:

[tex]y=ab^x[/tex]

Let x be the number of years and y be the allowance. The initial allowance is $8, this means at x = 0, y = 8

[tex]y=ab^x\\8=ab^0\\a=8[/tex]

Since it increases by 20% each year, i.e 100% + 20% = 1 + 0.2 = 1.2. This means that b = 1.2

Therefore:

[tex]y=ab^x\\y=8(1.2^x) \\[/tex]

To find the number of years will it take to reach $20 per month, we substitute y = 20 and find x

[tex]20=8(1.2)^x\\20/8=1.2^x\\1.2^x=2.5\\Taking \ natural\ log\ of \ both\ sides:\\ln(1.2^x)=ln2.5\\xln(1.2)=0.9163\\x=0.9163/ln(1.2)\\x=5.026[/tex]

x = 6 years to the nearest year

Answer:

5 years

Step-by-step explanation:444

coefficient of 8x+7y

Answers

Answer: I’m pretty sure it is 8 and 7 because those are the numbers multiplied with variables.

Answer:

8

Step-by-step explanation:

Identify the exponents on the variables in each term, and add them together to find the degree of each term.

8x→1

7y→1

The largest exponent is the degree of the polynomial.

1

The leading term in a polynomial is the term with the highest degree.

8x

The leading coefficient of a polynomial is the coefficient of the leading term.

____________________________________________________________

The leading term in a polynomial is the term with the highest degree.

8x

The leading coefficient in a polynomial is the coefficient of the leading term.

8

List the results.

Polynomial Degree: 1

Leading Term: 8x

Leading Coefficient: 8

Hope This Helps!!!

Determine if the process appears to be within statistical control. If not, state the reason why not.
a. It does not appear to be within statistical control because there is an upward shift.
b. It appears to be within statistical control.
c. It does not appear to be within statistical control because there is an upward trend.
d. It does not appear to be within statistical control because there is increasing variation.

Answers

Answer:

c. It does not appear to be within statistical control because there is an upward trend.

Step-by-step explanation:

Statistical process control is a method for quality control which employs statistical method to monitor and control process. It ensures operation efficiency and ensuring required specification to reduce wastes in production lines. Here the process variation is out of control because the statistical control has an upward trend.

A group of fitness club members lose a combined total of 28 kilograms in 1 week. There are approximately 2.2 pounds in 1 kilogram. Assuming the weight loss happened at a constant rate, about how many pounds did the group lose each day?

Answers

Answer:

8.8 pounds

Step-by-step explanation:

Given the following :

Combined weight loss which occurred within a week = 28 kg

Number of days in a week = 7 days

1 kilogram (kg) = 2.2 pounds

Combined weight loss in pounds that occurs within a week:

Weight loss in kg × 2.2

28kg * 2.2 = 61.6 pounds

Assume weight loss occurred at a constant rate :

Weight lost by the group per day :

(Total weight loss / number of days in a week)

(61.6 pounds / 7)

= 8.8 pounds daily

Answer:

88

Step-by-step explanation:

Found the answer and I am doing the quiz rn lel

WILL GIVE BRAINLEST PLEASE!!!!!!!! Jenny has some tiles in a bag. The tiles are of three different colors: purple, pink, and orange. Jenny randomly pulls a tile out of the bag, records the color, and replaces the tile in the bag. She does this 50 times. The results are recorded in the given table: Color of Tile Purple Pink Orange Number of times the tile is drawn 6 18 26 What is the experimental probability that Jenny will pull out a purple tile? fraction 6 over 50 fraction 44 over 50 fraction 6 over 44 fraction 18 over 44

Answers

Answer:

6/50

Step-by-step explanation:

There are 50 tiles

6 purple

18 pink

26 orange

P( purple) = purple/ total

                = 6/50

Compute each matrix sum or product if it is defined. If an expression is undefined. Explain why. Let A = (3 4 0 -4 -1 4), B = (8 1 -4 -5 2 -4), C = (1 -1 3 1) and D = (3 -2 4 5).

- 2A, B - 2A, AC, CD

Compute the matrix product -2A.

A. -2A =

B. The expression-2A is undefined because A is not a square matrix.

C. The expression-2A is undefined because matrices cannot be multiplied by numbers.

D. The expression 2A is undefined because matrices cannot have negative coefficients.

Answers

Answer:

-2A = (-6, -8, 0, 8, 2, -8)

B - 2A = (2, -7, -4, 3, 4, -12)

AC is undefined.

CD = (3, 2, 12, 5)

Step-by-step explanation:

Given the matrices:

A = (3 4 0 -4 -1 4)

B = (8 1 -4 -5 2 -4)

C = (1 -1 3 1)

D = (3 -2 4 5)

We are required to compute the following

-2A, B - 2A, AC, CD

For -2A:

-2(3 4 0 -4 -1 4)

= (-6, -8, 0, 8, 2, -8)

For B - 2A:

Because B - 2A = B + (-2A), we have:

(8 1 -4 -5 2 -4) + (-6, -8, 0, 8, 2, -8)

(2, -7, -4, 3, 4, -12)

For AC:

(3 4 0 -4 -1 4)(1 -1 3 1)

This is undefined.

For CD:

(1 -1 3 1)(3 -2 4 5)

= (3, 2, 12, 5)

Find the missing term in the
geometric sequence.
13,[ ? ],208

Answers

Answer:

110.5

Step-by-step explanation:

208=13+(3-1)d

208=13+2d

-13. -13

195=2d

÷2. ÷2

97.5=d. (d means difference)

13(first term)+97.5=110.5

Answer: 676

Step-by-step explanation: r/13=208/r

                                             r²=2704

                                               r=52

                                               13x52=676

                                               

find the area of this figure to the nearest hundredth. Use 3.14 to approximate pi.

Answers

Answer:

86.28 ft²

Step-by-step explanation:

The figure given consists of a rectangle and a semicircle.

The area of the figure = area of rectangle + area of semicircle

Area of rectangle = [tex] l*w [/tex]

Where,

l = 10 ft

w = 8 ft

[tex] area = l*w = 10*8 = 80 ft^2 [/tex]

Area of semicircle:

Area of semicircle = ½ of area of a circle = ½(πr²)

Where,

π = 3.14

r = ½ of 8 = 4 ft

Area of semi-circle = ½(3.14*4) = 6.28 ft²

Area of the figure = area of rectangle + area of semi-circle = 80 + 6.28 = 86.28 ft² (nearest hundredth)

Answer:

the area of the figrue is 105.12

Step-by-step explanation:

area of rectangle A= l · w10 x 8= 80area of simi-circle= 1/2(3.14 x r²)1/2 x 3.14 x 4²=25.1280+25.12=105.12 (nearest Hundredth)

Which expression is equal to (1+6i)−(7+3i) ?

Answers

Answer:

- 6+3i

Step-by-step explanation:

[tex](1+6i)-(7+3i) ?\\Group\:the\:real\:part\:and\:the\:imaginary\\\:part\:of\:the\:complex\:number\\\left(a+bi\right)\pm \left(c+di\right)=\left(a\:\pm \:c\right)+\left(b\:\pm \:d\right)i\\=\left(1-7\right)+\left(6-3\right)i\\1-7=-6\\6-3=3\\=-6+3i[/tex]

a lottery offers one $1000 prize one $500 and two $50 prizes. one thousand tickets are sold at $2.50. what is the expectived profit

Answers

Answer:

 $900

Step-by-step explanation:

To begin with let us estimate the total cash value of the prices

$1000 x 1= 1000

$500 x 1=  500

$50 x 2= 100

Total = $1600

Now let us calculate the total cost of tickets sold at $2.50 per tickets for 1000 tickets

2.5*1000= $2,500

Assuming worse case that the lottery had winners in all three categories and i.e the total prices given out is $1600

Then the expected profit is = $2,500-$1600= $900

Time spent using e-mail per session is normally distributed with a mean = to 8 minutes and standard deviation = 2minutes. If a random samples of 36 sessions were selected, the computed sample standard deviation would be

a. 0.25
b. 0.3333
c. 0.42
d. 0.48

Answers

Answer:

The correct option is (b) 0.3333.

Step-by-step explanation:

The standard deviation of the sampling distribution of sample mean [tex](\bar x)[/tex] is known as the standard error [tex](\sigma_{\bar x})[/tex].

The standard error is given as follows:

[tex]\sigma_{\bar x}=\frac{\sigma}{\sqrt{n}}[/tex]

The information provided is:

[tex]\mu=8\\\\\sigma=2\\\\n=36[/tex]

Compute the standard deviation of the sample mean as follows:

[tex]\sigma_{\bar x}=\frac{\sigma}{\sqrt{n}}[/tex]

    [tex]=\frac{2}{\sqrt{36}}\\\\=\frac{2}{6}\\\\=\frac{1}{3}\\\\=0.3333[/tex]

Thus, the standard deviation of the sample mean is 0.3333.

A paint machine dispenses dye into paint cans to create different shades of paint. The amount of dye dispensed into a can is known to have a normal distribution with a mean of 5 milliliters (ml) and a standard deviation of 0.4 ml. Answer the following questions based on this information. Find the dye amount that represents the 9th percentile of the distribution.

Answers

Answer:

4.464 ml

Step-by-step explanation:

Given that:

mean (μ) = 5 mm, standard deviation (σ) = 0.4 ml

The z score is a score in statistics used to determine by how many standard deviation the raw score is above or below the mean. If the z score is positive then the raw score is above the mean and if the z score is negative then the raw score is below the mean It is given by:

[tex]z=\frac{x-\mu}{\sigma}[/tex]

From the normal distribution table, the 9th percentile (0.09) corresponds to a z score of -1.34

[tex]z=\frac{x-\mu}{\sigma}\\\\-1.34=\frac{x-5}{0.4}\\\\x-5=-0.536\\\\x=5-0.536\\\\x=4.464[/tex]

The dye amount that represents the 9th percentile of the distribution is 4.464 ml

A new fast-food firm predicts that the number of franchises for its products will grow at the rate dn dt = 6 t + 1 where t is the number of years, 0 ≤ t ≤ 15.

Answers

Answer:

The answer is "253"

Step-by-step explanation:

In the given- equation there is mistype error so, the correct equation and its solution can be defined as follows:

Given:

[tex]\bold{\frac{dn}{dt} = 6\sqrt{t+1}}\\[/tex]

[tex]\to dn= 6\sqrt{t+1} \ \ dt.....(a)\\\\[/tex]

integrate the above value:

[tex]\to \int dn= \int 6\sqrt{t+1} \ \ dt \\\\\to n= \frac{(6\sqrt{t+1} )^{\frac{3}{2}}}{\frac{3}{2}}+c\\\\\to n= \frac{(12\sqrt{t+1} )^{\frac{3}{2}}}{3}+c\\\\[/tex]

When the value of n=1 then t=0

[tex]\to 1= \frac{12(0+1)^{\frac{3}{2}}}{3}+c\\\\ \to 1= \frac{12(1)^{\frac{3}{2}}}{3}+c\\\\\to 1-\frac{12}{3}=c\\\\\to \frac{3-12}{3}=c\\\\\to \frac{-9}{3}=c\\\\\to c=-3\\[/tex]

so the value of  n is:

[tex]\to n= \frac{(12\sqrt{t+1} )^{\frac{3}{2}}}{3}-3\\\\[/tex]

when we put the value t= 15 then,

[tex]\to n= \frac{(12\sqrt{15+1} )^{\frac{3}{2}}}{3}-3\\\\\to n= \frac{(12\sqrt{16} )^{\frac{3}{2}}}{3}-3\\\\\to n= \frac{(12\times 64)}{3}-3\\\\\to n= (4\times 64)-3\\\\\to n= 256-3\\\\\to n= 253[/tex]

Find an exact value of sin(17pi/12)

Answers

●✴︎✴︎✴︎✴︎✴︎✴︎✴︎✴︎❀✴︎✴︎✴︎✴︎✴︎✴︎✴︎✴︎✴︎●

   Hi my lil bunny!

❧⎯⎯⎯⎯⎯⎯⎯⎯⎯⎯⎯⎯⎯⎯⎯⎯⎯⎯⎯⎯⎯⎯⎯⎯⎯⎯⎯⎯⎯⎯⎯⎯⎯⎯⎯⎯⎯⎯☙

[tex]\frac{(17)(3.141593)}{12}[/tex]

= [tex]\frac{53.407075}{12}[/tex]

= [tex]4.45059[/tex]

❧⎯⎯⎯⎯⎯⎯⎯⎯⎯⎯⎯⎯⎯⎯⎯⎯⎯⎯⎯⎯⎯⎯⎯⎯⎯⎯⎯⎯⎯⎯⎯⎯⎯⎯⎯⎯⎯⎯☙

●✴︎✴︎✴︎✴︎✴︎✴︎✴︎✴︎❀✴︎✴︎✴︎✴︎✴︎✴︎✴︎✴︎✴︎●

If this helped you, could you maybe give brainliest..?

Also Have a great day/night!

❀*May*❀

I have an answer and explanation but I can't type so search up the question you asked and you should get an answer and explanation from s0cratic.

algebra pyramid please answer !! be the first to be marked as a brainliest .

Answers

Answer:

The first pyramid:

      63x

  39x  24x  

23x  16x  8x

The second pyramid:

     162x

 82x  80x  

4x  78x  2x

The third pyramid:

     12a+2b

9a+b   3a+b  

9a     b     3a

The fourth pyramid:

      -19a

  -5a   -14a  

3a   -8a  -6a

   

Step-by-step explanation:

All that an alegbra pyramid is is adding the two terms below it.

So you can see how I added the terms that lied below each number, such as in number 1:  I added 23x and 16x to get me 39x, and I added 16x and 8x to get me 24.

Hope this helped!

Find the indicated complement. A certain group of women has a 0.12​% rate of​ red/green color blindness. If a woman is randomly​ selected, what is the probability that she does not have​ red/green color​ blindness?

Answers

Answer:

the probability will be 0.

Step-by-step explanation:

0.12%= 0.0012= 3/2500.

Yelena needs to swim a total of 8 miles this
week. So far, she swam 5 miles. Use the
equation 5 + m=8 to find how many more
miles Yelena needs to swim.

Answers

Answer:

3 miles

Step-by-step explanation:

5 + m=8

Subtract 5 from each side

5-5 + m=8-5

m = 3

She needs to swim 3 more miles

Answer:

Yelena needs to swim 3 more miles

Step-by-step explanation:

You need to solve for the variable "m", which represents the miles. Based on the information, Yelena swam 5 miles and she needs to swim 8. Solve:

[tex]5+m=8[/tex]

To find the value of m, you need to isolate it on one side of the equation. To do this, you need to get the 8 and 5 on the same side of the equal operation. For this, you need to use reverse operations. This undoes the value from one side and does the same on the other, keeping the equation balanced. Since we have a "positive 5", we take the opposite, which would be a "negative 5". So subtract 5 from both sides of the equation:

[tex]5-5+m=8-5[/tex]

Simplify. The 5's cancel each other out, leaving 0. 8-5 is 3:

[tex]m=3[/tex]

The total miles left that Yelena needs to swim is 3 miles.

:Done

PLEASE HELP ! (2/5) -50 POINTS -

Answers

Answer:

symmetric

Step-by-step explanation:

it kind of evenly falls to the left and right from the highest value in the middle

skewed would be different and would look like a straight line not a quadratic equation

C) symmetric distribution because the mean, median, mode happen at about the same point

Find the measure of A.
A. 50
B. 70
C. 100
D. 90

Answers

Answer:

D

Step-by-step explanation:

I don't know how to eliminate the wrong answers.

Two line segments which have one end at a diameter and the other end meeting at a common point, make a 90 degree angle.

A is made that way, so A is 90 degrees.

Answer:

Step-by-step explanation:

I believe it is 90

What is the constant of variation, k, of the line y=kx through (3,18) and (5,30)? 3 6

Answers

Answer:

6

Step-by-step explanation:

The constant of variation is the slope

k = (y2-y1)/(x2-x1)

  = (30-18)/(5-3)

   =12/2

   = 6

The value of constant of variation, k, is,

⇒ k = 6

What is Equation of line?

The equation of line in point-slope form passing through the points

(x₁ , y₁) and (x₂, y₂) with slope m is defined as;

⇒ y - y₁ = m (x - x₁)

Where, m = (y₂ - y₁) / (x₂ - x₁)

Here, the constant of variation, k, of the line y = kx through (3,18) and (5,30)

Since, The constant of variation is the slope,

Hence, We get;

k = (y₂ - y₁)/(x₂ - x₁)

 = (30 - 18)/(5 - 3)

  = 12/2

  = 6

Thus, the value of constant of variation, k, is,

⇒ k = 6

Learn more about the equation of line visit:

https://brainly.com/question/18831322

#SPJ7

A cylindrical grain silo, with a flat top, is 30 feet tall and has a radius of 12 feet. It is full to the top with shelled corn. If the density of shelled corn averages 45 pounds/cubic foot, what does the corn in the silo weigh to the nearest pound

Answers

Answer:

610805 pounds

Step-by-step explanation:

The volume of grain in the silo will be calculated as equal to the volume of the cylinder formed by the silo

Height of the silo [tex]l[/tex] = 30 ft

radius of the silo r = 12 ft

volume of a cylinder = [tex]\pi r^{2} l[/tex]

substituting, we have

V = 3.142 x [tex]12^{2}[/tex] x 30 = 13573.44 cubic feet

We know that density ρ = weight/volume

density of the grains in the silo = 45 pound/cubic feet

therefore,

weight of grains = density x volume

weight of grains = 45 x 13573.44 = 610804.8 ≅ 610805 pounds

What is the measure of FEG?

A. 30 degrees

B. 40 degrees

C. 50 degrees

D. 70 degrees

Please include ALL work!! <3

Answers

Answer:

C. 50 degrees

Step-by-step explanation:

Because 6x + 5x = 110° and x = 10

5×10 = FEG 50°

Translate and solve: 54 greater than x is greater than 216

Answers

Answer:

x >162

Step-by-step explanation:

x+54 > 216

Subtract 54 from each side

x+54-54 > 216 - 54

x >162

Answer:

[tex]\huge \boxed{{x>162}}[/tex]

Step-by-step explanation:

[tex]x+54 > 216[/tex]

[tex]\sf Subtract \ 54 \ from \ both \ parts.[/tex]

[tex]x+54 -54> 216-54[/tex]

[tex]x>162[/tex]

Geometry pls help !!! Find the value of AB.
AB = [?]

Answers

Answer:

AB = 16 Units

Step-by-step explanation:

In the given figure, CD is the diameter and AB is the chord of the circle.

Since, diameter of the circle bisects the chord at right angle.

Therefore, AE = 1/2 AB

Or AB = 2AE...(1)

Let the center of the circle be given by O. Join OA.

OA = OD = 10 (Radii of same circle)

Triangle OAE is right triangle.

Now, by Pythagoras theorem:

[tex] OA^2 = AE^2 + OE^2 \\

10^2 = AE^2 + 6^2 \\

100= AE^2 + 36\\

100-36 = AE^2 \\

64= AE^2 \\

AE = \sqrt{64}\\

AE = 8 \\

\because AB = 2AE..[From \: equation\: (1)] \\

\therefore AB = 2\times 8\\

\huge \purple {\boxed {AB = 16 \: Units}} [/tex]

Other Questions
The joint density function for a pair of random variables X and Y is given. f(x, y) = Cx(1 + y) if 0 x 4, 0 y 4 0 otherwise f(x,y) = 0A) Find the value of the constant C. I already have 1/24.B) Find P(X < = 1, Y < = 1)C) Find P(X + Y < = 1). Originally, filters for Mr. Coffee electric-drip coffeemakers were sold through appliance and mass merchandise stores. If the company decided to sell the filters in grocery stores where customers purchased their coffee, what type of intermediaries would the company most likely hire which approach does procedural programming follow? bottom up, top down, random, or object oriented It seems almost unbelievable that so many students are sick during midterms and final exams, but actually these are times of _________ stress that _________ the effectiveness of the immune system to fight off illness. Group of answer choices Please help ! First one to give correct answer gets brainliest! Though not specifically cited in the producer's contract, the producer is expected to telephone prospects on the insurer's behalf to arrange sales appointments. This is an example of what kind of producer authority? A share of stock is now selling for $110. It will pay a dividend of $8 per share at the end of the year. Its beta is 1. What do investors expect the stock to sell for at the end of the year? Assume the risk-free rate is 4% and the expected rate of return on the market is 15%. (Round your answer to 2 decimal places.) Expected selling price $ Help a friend out I dont understand it what is the lcm of 725 and 325 The three-dimensional figure shown consists of a cylinder and a right circular cone. The radius of the base is 10 centimeters. The height of the cylinder is 16 centimeters, and the total height of the figure is 28 centimeters. The slant height of the cone is 13 centimeters. Which choice is the best approximation of the surface area of the figure? Use 3.14 to approximate pi. Molar mass is reported on the periodic table in the unitsA. kg/m3B. grams/mole.C. moles/gram.D. atoms/gram. Examples of innovation,creativity and entrepreneurship What caused problems during Grant's presidency?O A. ReconstructionO B. Corrupt staff membersO C. The Civil WarXO D. Grant's own dishonesty What is the definition of czar. Vector has a magnitude of 6.0 m and points 30 north of east. Vector has a magnitude of 4.0 m and points 30 east of north. The resultant vector + is given by What is the proper way to form the negative t command "don't buy it?"O A. No lo compres.O B. No lo compras.O C. No lo compraO D. No lo compris. Question 15 please and i will mark the brainliest!!! And thank you to whoever answers Which cause-and-effect diagram best illustrates one main effect of informedcitizens on the political system?A.Citizens educatethemselves onimportant issuesPoliticians cancommunicate theirplans to citizensmore easilyOBCitizens read newssources they agreewith.These citizensbecome moreengaged in politics.11O c.Citizens understandhow to acquireInformationCitizens are morelikely to know howto vote.OD.Citizens analyze theactions and words ofelected officials.officials are held toaccount when theymisbehavePREVIOUS An employer plans to pay bonus to each of his employees. Those earning Rs. 2000 or above are to be paid 10 percent of their salary and those earning less than 2000 are to be paid Rs. 200. The input record contains employee number, name and salary of an employee. The output should be employee name and their bonus. Write pseudo code algorithm to process the requirement. if b is the midpoint of ac and if c is the midpoint of bd, then what percent of cd is ac